Does artificial gravity based on centrifugal force stop working if you jump off the ground?











up vote
20
down vote

favorite
1












In an answer to another question of mine, concerning gravity, there was a link to a video about creation of artificial gravity, based on rotation.



The question I have might be silly (or with an obvious answer), but it puzzles me non the less. As I understand it, in order for the centrifugal force (which is responsible for creating gravity, in this case) to work, object it works upon should be attached to the wheels 'spoke' or 'rim'. If an astronaut walks on the inside of the 'rim' (like here in the video), the contact with the 'rim' is maintained via legs, thus centrifugal force is in action.



Now, the question: if, while being inside a rotating space station, astronaut would jump really high, wouldn't he then experience zero gravity until he again will touch some part (wall or floor) of the station? Am I missing something in my understanding?










share|cite|improve this question




















  • 17




    For an experimental exploration of answers to this question, take three friends and a dodgeball to your nearest playground with a merry-go-round. Get the merry-go-round up to speed, then have the folks who are riding it try to play "catch" with the dodgeball. Take turns observing from off the merry-go-round to see the difference between the rotating frame and the inertial frame. What happens is quite surprising, even if you make a prediction on paper beforehand, because the intuition you use to play "catch" is not well-adapted to rotating reference frames.
    – rob
    yesterday






  • 2




    It seems to work just fine. Space Station Centrifuge Gravity Simulation 196x NASA color 3min The word jump needs to be in the title.
    – Mazura
    yesterday






  • 5




    While jumping may remove you from the force imparted by the "floor" of the rotating drum, consider that you have momentum and will continue to move. The direction of this movement will be tangent to the curve of the rotating drum in the direction of rotation. So while you will not "fall" straight back down to the floor, you will drift forwards with your current momentum and... bump right back into the floor.
    – Blackhawk
    yesterday






  • 1




    @Anoplexian if the acceleration is temporarily zero, the velocity will be temporarily constant. I think you're speaking of the moment at which the velocity is temporarily zero, which is not the same as the acceleration being temporarily zero.
    – phoog
    18 hours ago






  • 1




    Technically, if you're standing here on earth and you jump really high, then you'll experience zero gravity, too. 😉
    – David Richerby
    18 hours ago

















up vote
20
down vote

favorite
1












In an answer to another question of mine, concerning gravity, there was a link to a video about creation of artificial gravity, based on rotation.



The question I have might be silly (or with an obvious answer), but it puzzles me non the less. As I understand it, in order for the centrifugal force (which is responsible for creating gravity, in this case) to work, object it works upon should be attached to the wheels 'spoke' or 'rim'. If an astronaut walks on the inside of the 'rim' (like here in the video), the contact with the 'rim' is maintained via legs, thus centrifugal force is in action.



Now, the question: if, while being inside a rotating space station, astronaut would jump really high, wouldn't he then experience zero gravity until he again will touch some part (wall or floor) of the station? Am I missing something in my understanding?










share|cite|improve this question




















  • 17




    For an experimental exploration of answers to this question, take three friends and a dodgeball to your nearest playground with a merry-go-round. Get the merry-go-round up to speed, then have the folks who are riding it try to play "catch" with the dodgeball. Take turns observing from off the merry-go-round to see the difference between the rotating frame and the inertial frame. What happens is quite surprising, even if you make a prediction on paper beforehand, because the intuition you use to play "catch" is not well-adapted to rotating reference frames.
    – rob
    yesterday






  • 2




    It seems to work just fine. Space Station Centrifuge Gravity Simulation 196x NASA color 3min The word jump needs to be in the title.
    – Mazura
    yesterday






  • 5




    While jumping may remove you from the force imparted by the "floor" of the rotating drum, consider that you have momentum and will continue to move. The direction of this movement will be tangent to the curve of the rotating drum in the direction of rotation. So while you will not "fall" straight back down to the floor, you will drift forwards with your current momentum and... bump right back into the floor.
    – Blackhawk
    yesterday






  • 1




    @Anoplexian if the acceleration is temporarily zero, the velocity will be temporarily constant. I think you're speaking of the moment at which the velocity is temporarily zero, which is not the same as the acceleration being temporarily zero.
    – phoog
    18 hours ago






  • 1




    Technically, if you're standing here on earth and you jump really high, then you'll experience zero gravity, too. 😉
    – David Richerby
    18 hours ago















up vote
20
down vote

favorite
1









up vote
20
down vote

favorite
1






1





In an answer to another question of mine, concerning gravity, there was a link to a video about creation of artificial gravity, based on rotation.



The question I have might be silly (or with an obvious answer), but it puzzles me non the less. As I understand it, in order for the centrifugal force (which is responsible for creating gravity, in this case) to work, object it works upon should be attached to the wheels 'spoke' or 'rim'. If an astronaut walks on the inside of the 'rim' (like here in the video), the contact with the 'rim' is maintained via legs, thus centrifugal force is in action.



Now, the question: if, while being inside a rotating space station, astronaut would jump really high, wouldn't he then experience zero gravity until he again will touch some part (wall or floor) of the station? Am I missing something in my understanding?










share|cite|improve this question















In an answer to another question of mine, concerning gravity, there was a link to a video about creation of artificial gravity, based on rotation.



The question I have might be silly (or with an obvious answer), but it puzzles me non the less. As I understand it, in order for the centrifugal force (which is responsible for creating gravity, in this case) to work, object it works upon should be attached to the wheels 'spoke' or 'rim'. If an astronaut walks on the inside of the 'rim' (like here in the video), the contact with the 'rim' is maintained via legs, thus centrifugal force is in action.



Now, the question: if, while being inside a rotating space station, astronaut would jump really high, wouldn't he then experience zero gravity until he again will touch some part (wall or floor) of the station? Am I missing something in my understanding?







newtonian-mechanics forces reference-frames free-body-diagram centrifugal-force






share|cite|improve this question















share|cite|improve this question













share|cite|improve this question




share|cite|improve this question








edited 16 hours ago









knzhou

39.7k9111194




39.7k9111194










asked yesterday









Filipp W.

24616




24616








  • 17




    For an experimental exploration of answers to this question, take three friends and a dodgeball to your nearest playground with a merry-go-round. Get the merry-go-round up to speed, then have the folks who are riding it try to play "catch" with the dodgeball. Take turns observing from off the merry-go-round to see the difference between the rotating frame and the inertial frame. What happens is quite surprising, even if you make a prediction on paper beforehand, because the intuition you use to play "catch" is not well-adapted to rotating reference frames.
    – rob
    yesterday






  • 2




    It seems to work just fine. Space Station Centrifuge Gravity Simulation 196x NASA color 3min The word jump needs to be in the title.
    – Mazura
    yesterday






  • 5




    While jumping may remove you from the force imparted by the "floor" of the rotating drum, consider that you have momentum and will continue to move. The direction of this movement will be tangent to the curve of the rotating drum in the direction of rotation. So while you will not "fall" straight back down to the floor, you will drift forwards with your current momentum and... bump right back into the floor.
    – Blackhawk
    yesterday






  • 1




    @Anoplexian if the acceleration is temporarily zero, the velocity will be temporarily constant. I think you're speaking of the moment at which the velocity is temporarily zero, which is not the same as the acceleration being temporarily zero.
    – phoog
    18 hours ago






  • 1




    Technically, if you're standing here on earth and you jump really high, then you'll experience zero gravity, too. 😉
    – David Richerby
    18 hours ago
















  • 17




    For an experimental exploration of answers to this question, take three friends and a dodgeball to your nearest playground with a merry-go-round. Get the merry-go-round up to speed, then have the folks who are riding it try to play "catch" with the dodgeball. Take turns observing from off the merry-go-round to see the difference between the rotating frame and the inertial frame. What happens is quite surprising, even if you make a prediction on paper beforehand, because the intuition you use to play "catch" is not well-adapted to rotating reference frames.
    – rob
    yesterday






  • 2




    It seems to work just fine. Space Station Centrifuge Gravity Simulation 196x NASA color 3min The word jump needs to be in the title.
    – Mazura
    yesterday






  • 5




    While jumping may remove you from the force imparted by the "floor" of the rotating drum, consider that you have momentum and will continue to move. The direction of this movement will be tangent to the curve of the rotating drum in the direction of rotation. So while you will not "fall" straight back down to the floor, you will drift forwards with your current momentum and... bump right back into the floor.
    – Blackhawk
    yesterday






  • 1




    @Anoplexian if the acceleration is temporarily zero, the velocity will be temporarily constant. I think you're speaking of the moment at which the velocity is temporarily zero, which is not the same as the acceleration being temporarily zero.
    – phoog
    18 hours ago






  • 1




    Technically, if you're standing here on earth and you jump really high, then you'll experience zero gravity, too. 😉
    – David Richerby
    18 hours ago










17




17




For an experimental exploration of answers to this question, take three friends and a dodgeball to your nearest playground with a merry-go-round. Get the merry-go-round up to speed, then have the folks who are riding it try to play "catch" with the dodgeball. Take turns observing from off the merry-go-round to see the difference between the rotating frame and the inertial frame. What happens is quite surprising, even if you make a prediction on paper beforehand, because the intuition you use to play "catch" is not well-adapted to rotating reference frames.
– rob
yesterday




For an experimental exploration of answers to this question, take three friends and a dodgeball to your nearest playground with a merry-go-round. Get the merry-go-round up to speed, then have the folks who are riding it try to play "catch" with the dodgeball. Take turns observing from off the merry-go-round to see the difference between the rotating frame and the inertial frame. What happens is quite surprising, even if you make a prediction on paper beforehand, because the intuition you use to play "catch" is not well-adapted to rotating reference frames.
– rob
yesterday




2




2




It seems to work just fine. Space Station Centrifuge Gravity Simulation 196x NASA color 3min The word jump needs to be in the title.
– Mazura
yesterday




It seems to work just fine. Space Station Centrifuge Gravity Simulation 196x NASA color 3min The word jump needs to be in the title.
– Mazura
yesterday




5




5




While jumping may remove you from the force imparted by the "floor" of the rotating drum, consider that you have momentum and will continue to move. The direction of this movement will be tangent to the curve of the rotating drum in the direction of rotation. So while you will not "fall" straight back down to the floor, you will drift forwards with your current momentum and... bump right back into the floor.
– Blackhawk
yesterday




While jumping may remove you from the force imparted by the "floor" of the rotating drum, consider that you have momentum and will continue to move. The direction of this movement will be tangent to the curve of the rotating drum in the direction of rotation. So while you will not "fall" straight back down to the floor, you will drift forwards with your current momentum and... bump right back into the floor.
– Blackhawk
yesterday




1




1




@Anoplexian if the acceleration is temporarily zero, the velocity will be temporarily constant. I think you're speaking of the moment at which the velocity is temporarily zero, which is not the same as the acceleration being temporarily zero.
– phoog
18 hours ago




@Anoplexian if the acceleration is temporarily zero, the velocity will be temporarily constant. I think you're speaking of the moment at which the velocity is temporarily zero, which is not the same as the acceleration being temporarily zero.
– phoog
18 hours ago




1




1




Technically, if you're standing here on earth and you jump really high, then you'll experience zero gravity, too. 😉
– David Richerby
18 hours ago






Technically, if you're standing here on earth and you jump really high, then you'll experience zero gravity, too. 😉
– David Richerby
18 hours ago












8 Answers
8






active

oldest

votes

















up vote
40
down vote














Now, the question: if, while being inside a rotating space station, astronaut would jump really high, wouldn't he then experience zero gravity until he again will touch some part (wall or floor) of the station? Am I missing something in my understanding?




Well, here's a related question. Suppose you find yourself in an elevator at the top floor of a skyscraper when the cable suddenly snaps. As the elevator plummets down, you realize you'll die on impact when it reaches the bottom. But then you think, what if I jump just before that happens? When you jump, you're moving up, not down, so there won't be any impact at all!



The mistake here is the same as the one you're made above. When you jump in the elevator, you indeed start moving upward relative to the elevator, but you're still moving at a tremendous speed downward relative to the ground, which is what matters.



Similarly, when you are at the rim of a large rotating space station, you have a large velocity relative to somebody standing still at the center. When you jump, it's true that you're going up relative to the piece of ground you jumped from, but you still have that huge rotational velocity. You don't lose it just by losing contact with the ground, so nothing about the story changes.






share|cite|improve this answer

















  • 5




    Another, related example: A hovering helicopter on earth does not see the earth move underneath it with 1000 km/h or so (due to earth rotation), simply because that same helicopter was moving at the same speed with the earth all along.
    – user1583209
    yesterday






  • 2




    Please do correct me if I am wrong, but I think here what happens in your explanation is that although you do land on the same spot of the disc after jumping off, it’s less due to the efffect of gravity and more due to the ring rotating to ‘catch’ you. Although you do have a horizontal velocity, when you jump off I think you don’t actually move in a circle, rather you move in a straight line until hitting the ring again. In that sense, OP is technically correct since you aren’t really(assumed gravity negligible) under the effect of some centrifugal force when you jump?
    – EigenFunction
    yesterday






  • 6




    @EigenFunction You definitely are under such a force in the rotating frame of reference of the rim of the station. True, an external observer would see you move in a straight line and intersect the rim again with no centrifugal force involved, but that observer never sees a centrifugal force--only the centripetal normal force of the station's rim on your feet.
    – eyeballfrog
    yesterday








  • 7




    @EigenFunction You could always say that, though. For example, you could say that gravity on Earth doesn't really exist: a thrown ball always goes in a straight line, but the floor is just constantly accelerating up to catch it. The fact that this is always just as good a description is the content of the equivalence principle, i.e. the foundation of general relativity. You just can't tell the two apart.
    – knzhou
    yesterday








  • 1




    @simple if you enter the spinning room from an axle having not touched the station you would indeed just float there while the station spins around you. If you enter that room from a rotating spoke you'll find you'll also float but you're body will spin as you float because you'll have the same angular momentum you had when you last touched the station.
    – candied_orange
    22 hours ago




















up vote
17
down vote













If you jump then you are in free fall, apart from air resistance, so you are weightless. This holds for any jump. For a brief moment you experience zero gravity!






share|cite|improve this answer

















  • 3




    You may add that unfortunately, your environment continues to move around you. (This is true even when on earth, because the gravity exerted by our planet is indistinguishable from an accelerated movement of our planet towards us. (Hello to all flat-earthers!)
    – Peter A. Schneider
    15 hours ago












  • On the other hand you could run opposite to the direction of the ships rotation, and find yourself in lower and lower gravity as you went faster. going at the same angular velocity you would indeed find yourself weightless.
    – R. Rankin
    6 hours ago


















up vote
3
down vote













This is an inertia problem. The phrasing of your question presupposes that the astronaut loses inertia when jumping, resulting in their upward trajectory being the only major acting force. At any point while rotating in a stationary position, the inertia of the astronaut is outwards from the rotational path at a 90 degree angle from the line connecting the astronaut and the axis of rotation. When the astronaut jumps, this inertia is maintained and two new forces are generated, namely the force the astronaut enacts on the space station (resulting in negligible acceleration on the part of the space station) and the force the space station enacts on the astronaut (resulting in non-negligible acceleration on the part of the astronaut towards the axis of rotation). The astronaut's relative velocity is related linearly to the vector sum of the inertial force acting on him as well as the force from his jump. This is why the "gravity" felt by the astronaut is determined by the angular velocity of the station floor; greater angular velocity means greater inertia as well as greater centripetal (and centrifugal) force, which means greater force (applied by jumping) is required to accomplish the same (relatively) vertical displacement.



The feeling of being pressed to the ground in an artificial gravity chamber is due to the rotating floor pushing the astronaut towards the rotational axis, when the astronaut (from a physics perspective) wants to keep going along the inertial path. This force resulting in this central acceleration is the centripetal force, while the opposing force enacted by the astronaut on the space station floor is the centrifugal force.



You can find this information in any basic university level physics book; pay special attention to chapters covering angular momentum and gravitational/circular forces. Fundamentals of Physics by David Halliday and Robert Resnick was the one I used in college.






share|cite|improve this answer








New contributor




SweepingsDemon is a new contributor to this site. Take care in asking for clarification, commenting, and answering.
Check out our Code of Conduct.

























    up vote
    1
    down vote













    Of course, the biggest problem with the supposition of becoming 'weightless' (returning to a null-G state) by making a 'big' jump is this: Assuming the OP was talking about not a created micro-gravity (.2G or less) but closer to one full gravity (Earth standard), is the size of the 'room' required to create that much tidal force. The smaller it is, the faster it would have to be spinning to create sufficient centripetal force to give the impression of that 'gravity', and subsequently the thinner the band of that 'gravity' as the further from the zone of that 'gravity' the less the force.



    So unless it were some silly, tiny spinning room, the size of the 'station' required to create a 'natural gravitational' feel would far exceed the astronaut's ability to jump without outside propulsion. In that case, of course, the ability to then travel outside the functional 'gravity' into decreasingly accelerated zones (by removing his own laterally imparted velocity) he could then eventually reach the 0G center of the station or room.



    However, with the additional concept of scale, the same could be said of the Earth itself. If you could 'jump' to a 'high enough' height, you could reach 0G (not really, but sufficient micro gravity as to be indistinguishable from 0G). This does give pause for thought, as not only would such a station need to be QUITE large, but that you'd better be wearing a low-pressure suit at the same time, as the atmosphere should be quite thin, since the Nitrogen/Oxygen atmosphere would ALSO be affected by the centripetal force of the station's rotation, thus not only be thin, but definitely not made up of the proper gasses for breathing (lighter gasses rising into the center of spin)... At least that's what I'd expect.






    share|cite|improve this answer








    New contributor




    Asuka Jr. is a new contributor to this site. Take care in asking for clarification, commenting, and answering.
    Check out our Code of Conduct.

























      up vote
      1
      down vote














      As I understand it, in order for the centrifugal force (which is responsible for creating gravity, in this case) to work, object it works upon should be attached to the wheels 'spoke' or 'rim'.




      Not exactly. Centrifugal "force" is what's known as a "pseudo-force". It's the result of analyzing events using a non-inertial reference frame. If you're on a merry-go-round, and you treat the merry-go-round as being stationary while the world revolves around it, you will find that objects have a tendency to go towards the outside of the merry-go-round. So within the "The merry-go-round is stationary" point view, you have to posit some force pushing the objects away from the center, which is the centrifugal force.



      But this force doesn't "really" exist: when you analyze the situation from the point of view of someone not on the merry-go-round, the objects are traveling in a straight line. It's just that any straight line will necessarily go away from the center. (Draw a circle, then draw straight line next to it. Imagine traveling along that line. From the point of view of the circle, you're first getting closer, then traveling away.)



      When something is rotating, its velocity is constantly changing: although its speed is constant, the direction is changing, so the velocity is changing. Changing velocity means acceleration, and acceleration means force. This force is directed towards the center. Imagine driving around a circle counterclockwise. If you were to let go of the wheel, you would fly off the circle. You have to constantly turn left to stay on the circle. So there is a force, but it's towards the center of the circle, and is called the centripetal force.



      From an inertial reference frame pint of view, a force is needed to stay on the circle; the centripetal force. But from a circular motion reference frame, the object is stationary. So if there's a centripetal force pulling the object in, there must be another force, the centrifugal force, pushing it out. So if you're standing in a rotating space station, you'll going to feel a force of the floor pushing you "up" towards the center of the space station, and since it feels like you're at rest (the space station is moving with you), it's going to seem that there must be some force pushing you "down" into the floor.



      The important point here is that the contact with the floor provides the centripetal force, but the centrifugal force exists in your reference frame regardless of whether you have contact with the floor. Go back to the example of driving in a circle. Suppose you drop a ball in the car. Before you dropped it, it was moving with the car, and so just as the car had a centripetal force keeping in circular motion, the ball had a centripetal force on it. But for the fraction of a second that it's in the air, it doesn't have the centripetal force.



      For an outside observer, the car is turning left, while the ball is moving in a straight line. The car accelerates to the left into the ball, and when the ball lands, it is to the right of where it was dropped. For someone in the car, however, it seems like the car is stationary, and ball is accelerating to the right.



      Similarly, if you were to jump in the space station, then since you are perceiving things in the space station's reference frame, it will seem like you are accelerating towards the floor. This apparent acceleration exists regardless of whether you're touching the space station. That you are accelerating in the station's reference frame doesn't require physical contact with the station because it's not a physical phenomenon. It's simply an attribute of the coordinate system.



      All of this applies locally: if you jump up, your motion will, in the station's reference frame, on small scales be the same as if you were being pulled down by gravity. This is an approximation that breaks down as you go to larger scales. These deviations from the approximation show up as other pseudo-forces, such as the Coriolis force. So being in contact with the floor does matter in that it keeps you moving with the station and reduces these deviations.






      share|cite|improve this answer





















      • Maybe worth emphasising the Coriolis force a bit more, and describing the case where someone jumps backwards fast enough to cancel the rotation and so the centrifugal force would still exist but would be exactly balanced by coriolis so they would appear to fly above the floor without falling. (until they hit a wall)
        – bdsl
        10 hours ago


















      up vote
      0
      down vote













      You're not making any mistake except for thinking "artificial gravity" could be
      nearly constant over a region as big as the structure itself. It really always
      applies only to a region "much smaller" than the structure itself.



      So, yes, a big jump (up and backwards on the wheel) could send you through the
      middle of the wheel, where you would just float. Or, more simply, running fast
      enough (backwards on the wheel) will cause you to levitate.



      This non-constant variation of your "artificial gravity" in spacetime was already
      explained as a "tidal force" in my2cts's comment.






      share|cite|improve this answer




























        up vote
        0
        down vote













        Here's a really simple experiment you can perform right now.



        You are standing on the surface of the Earth which is rotating at, depending on your latitude, somewhere between 1000 and 1600 km/h. If you dare, stand up and jump. Did you suddenly hurdle sideways at a great speed? No. Similarly a lamp hanging from the ceiling isn't thrown violently to one the side despite not touching the ground.



        Your momentum is conserved. And momentum is mass x velocity. Your mass doesn't change when you jump, so we can just talk about conserving velocity.



        Velocity is a vector meaning it has a magnitude and a direction. Standing still you're traveling at, say, 1200 km/h going sideways, but so is the surface of the Earth and your house and the air so you don't perceive it. When you jump you retain that 1200 km/h sideways velocity and add about 2m/s upward velocity. Since everything else is moving sideways with you, all you perceive is the vertical jump.



        But only your linear momentum is conserved. Gravity is keeping you stuck to the rotating Earth. If you were to somehow nullify Earth's gravitational pull you'd find yourself slowly, and then more and more rapidly, seeming to rise up from the surface of the Earth. Your linear momentum is carrying you along in a straight line, but the surface of the Earth is curved. It seems like the surface is dropping away, but without gravity sticking you to the surface you'll travel in a straight line over a curved surface.



        This difference between the linear and angular momentum is the Coriolis force and has practical effects on Earth and on a space station. On Earth it's why storms swirl. The much smaller scale on a space station can have subtle effects if you throw a ball, shoot a gun, or even on your inner ear.






        share|cite|improve this answer




























          up vote
          0
          down vote













          Just a simplified example scenario depicting what the angular velocity answers say:



          Assume a large open cylinder set up as a jogging track that is an entire rotating "Ring" of the a space station.



          Instead of jumping, run as fast as the track section is rotating but in the opposite direction. You will appear to lose weight as you accelerate since you will no longer have the angular momentum of the floor but something less than that.



          When you reach the speed the station is spinning, you will be weightless and lift off the ground (if you don't lose traction first). While above the ground in this way you will be weightless.



          This also means, intuitively, that if you ran with the spin of the station you would get heavier.



          Once you see it this way, you can see that jumping wouldn't really do anything in itself.



          tl;dr



          Problem 1:
          After you become weightless the air is probably rotating with the floor of the track, so it will push your body giving it some angular momentum again at which point you will be "Pulled" back to the surface. If being able to alter your weight was a partial goal of this exercise room they could make the floor and walls fairly smooth so that the air stayed (mostly) in place and ignored the spin altogether. If this were the case you'd start with a wind at your back, but the faster you ran the weaker the wind would become and when you became weightless there would be no air pushing you.



          Problem 2: In order to run the speed of the station the starting gravity may have to be less than 1g, Not sure of the math at all, but I am sure with the right starting speed you could run until you were weightless.






          share|cite|improve this answer























            Your Answer





            StackExchange.ifUsing("editor", function () {
            return StackExchange.using("mathjaxEditing", function () {
            StackExchange.MarkdownEditor.creationCallbacks.add(function (editor, postfix) {
            StackExchange.mathjaxEditing.prepareWmdForMathJax(editor, postfix, [["$", "$"], ["\\(","\\)"]]);
            });
            });
            }, "mathjax-editing");

            StackExchange.ready(function() {
            var channelOptions = {
            tags: "".split(" "),
            id: "151"
            };
            initTagRenderer("".split(" "), "".split(" "), channelOptions);

            StackExchange.using("externalEditor", function() {
            // Have to fire editor after snippets, if snippets enabled
            if (StackExchange.settings.snippets.snippetsEnabled) {
            StackExchange.using("snippets", function() {
            createEditor();
            });
            }
            else {
            createEditor();
            }
            });

            function createEditor() {
            StackExchange.prepareEditor({
            heartbeatType: 'answer',
            convertImagesToLinks: false,
            noModals: true,
            showLowRepImageUploadWarning: true,
            reputationToPostImages: null,
            bindNavPrevention: true,
            postfix: "",
            imageUploader: {
            brandingHtml: "Powered by u003ca class="icon-imgur-white" href="https://imgur.com/"u003eu003c/au003e",
            contentPolicyHtml: "User contributions licensed under u003ca href="https://creativecommons.org/licenses/by-sa/3.0/"u003ecc by-sa 3.0 with attribution requiredu003c/au003e u003ca href="https://stackoverflow.com/legal/content-policy"u003e(content policy)u003c/au003e",
            allowUrls: true
            },
            noCode: true, onDemand: true,
            discardSelector: ".discard-answer"
            ,immediatelyShowMarkdownHelp:true
            });


            }
            });














            draft saved

            draft discarded


















            StackExchange.ready(
            function () {
            StackExchange.openid.initPostLogin('.new-post-login', 'https%3a%2f%2fphysics.stackexchange.com%2fquestions%2f446889%2fdoes-artificial-gravity-based-on-centrifugal-force-stop-working-if-you-jump-off%23new-answer', 'question_page');
            }
            );

            Post as a guest















            Required, but never shown

























            8 Answers
            8






            active

            oldest

            votes








            8 Answers
            8






            active

            oldest

            votes









            active

            oldest

            votes






            active

            oldest

            votes








            up vote
            40
            down vote














            Now, the question: if, while being inside a rotating space station, astronaut would jump really high, wouldn't he then experience zero gravity until he again will touch some part (wall or floor) of the station? Am I missing something in my understanding?




            Well, here's a related question. Suppose you find yourself in an elevator at the top floor of a skyscraper when the cable suddenly snaps. As the elevator plummets down, you realize you'll die on impact when it reaches the bottom. But then you think, what if I jump just before that happens? When you jump, you're moving up, not down, so there won't be any impact at all!



            The mistake here is the same as the one you're made above. When you jump in the elevator, you indeed start moving upward relative to the elevator, but you're still moving at a tremendous speed downward relative to the ground, which is what matters.



            Similarly, when you are at the rim of a large rotating space station, you have a large velocity relative to somebody standing still at the center. When you jump, it's true that you're going up relative to the piece of ground you jumped from, but you still have that huge rotational velocity. You don't lose it just by losing contact with the ground, so nothing about the story changes.






            share|cite|improve this answer

















            • 5




              Another, related example: A hovering helicopter on earth does not see the earth move underneath it with 1000 km/h or so (due to earth rotation), simply because that same helicopter was moving at the same speed with the earth all along.
              – user1583209
              yesterday






            • 2




              Please do correct me if I am wrong, but I think here what happens in your explanation is that although you do land on the same spot of the disc after jumping off, it’s less due to the efffect of gravity and more due to the ring rotating to ‘catch’ you. Although you do have a horizontal velocity, when you jump off I think you don’t actually move in a circle, rather you move in a straight line until hitting the ring again. In that sense, OP is technically correct since you aren’t really(assumed gravity negligible) under the effect of some centrifugal force when you jump?
              – EigenFunction
              yesterday






            • 6




              @EigenFunction You definitely are under such a force in the rotating frame of reference of the rim of the station. True, an external observer would see you move in a straight line and intersect the rim again with no centrifugal force involved, but that observer never sees a centrifugal force--only the centripetal normal force of the station's rim on your feet.
              – eyeballfrog
              yesterday








            • 7




              @EigenFunction You could always say that, though. For example, you could say that gravity on Earth doesn't really exist: a thrown ball always goes in a straight line, but the floor is just constantly accelerating up to catch it. The fact that this is always just as good a description is the content of the equivalence principle, i.e. the foundation of general relativity. You just can't tell the two apart.
              – knzhou
              yesterday








            • 1




              @simple if you enter the spinning room from an axle having not touched the station you would indeed just float there while the station spins around you. If you enter that room from a rotating spoke you'll find you'll also float but you're body will spin as you float because you'll have the same angular momentum you had when you last touched the station.
              – candied_orange
              22 hours ago

















            up vote
            40
            down vote














            Now, the question: if, while being inside a rotating space station, astronaut would jump really high, wouldn't he then experience zero gravity until he again will touch some part (wall or floor) of the station? Am I missing something in my understanding?




            Well, here's a related question. Suppose you find yourself in an elevator at the top floor of a skyscraper when the cable suddenly snaps. As the elevator plummets down, you realize you'll die on impact when it reaches the bottom. But then you think, what if I jump just before that happens? When you jump, you're moving up, not down, so there won't be any impact at all!



            The mistake here is the same as the one you're made above. When you jump in the elevator, you indeed start moving upward relative to the elevator, but you're still moving at a tremendous speed downward relative to the ground, which is what matters.



            Similarly, when you are at the rim of a large rotating space station, you have a large velocity relative to somebody standing still at the center. When you jump, it's true that you're going up relative to the piece of ground you jumped from, but you still have that huge rotational velocity. You don't lose it just by losing contact with the ground, so nothing about the story changes.






            share|cite|improve this answer

















            • 5




              Another, related example: A hovering helicopter on earth does not see the earth move underneath it with 1000 km/h or so (due to earth rotation), simply because that same helicopter was moving at the same speed with the earth all along.
              – user1583209
              yesterday






            • 2




              Please do correct me if I am wrong, but I think here what happens in your explanation is that although you do land on the same spot of the disc after jumping off, it’s less due to the efffect of gravity and more due to the ring rotating to ‘catch’ you. Although you do have a horizontal velocity, when you jump off I think you don’t actually move in a circle, rather you move in a straight line until hitting the ring again. In that sense, OP is technically correct since you aren’t really(assumed gravity negligible) under the effect of some centrifugal force when you jump?
              – EigenFunction
              yesterday






            • 6




              @EigenFunction You definitely are under such a force in the rotating frame of reference of the rim of the station. True, an external observer would see you move in a straight line and intersect the rim again with no centrifugal force involved, but that observer never sees a centrifugal force--only the centripetal normal force of the station's rim on your feet.
              – eyeballfrog
              yesterday








            • 7




              @EigenFunction You could always say that, though. For example, you could say that gravity on Earth doesn't really exist: a thrown ball always goes in a straight line, but the floor is just constantly accelerating up to catch it. The fact that this is always just as good a description is the content of the equivalence principle, i.e. the foundation of general relativity. You just can't tell the two apart.
              – knzhou
              yesterday








            • 1




              @simple if you enter the spinning room from an axle having not touched the station you would indeed just float there while the station spins around you. If you enter that room from a rotating spoke you'll find you'll also float but you're body will spin as you float because you'll have the same angular momentum you had when you last touched the station.
              – candied_orange
              22 hours ago















            up vote
            40
            down vote










            up vote
            40
            down vote










            Now, the question: if, while being inside a rotating space station, astronaut would jump really high, wouldn't he then experience zero gravity until he again will touch some part (wall or floor) of the station? Am I missing something in my understanding?




            Well, here's a related question. Suppose you find yourself in an elevator at the top floor of a skyscraper when the cable suddenly snaps. As the elevator plummets down, you realize you'll die on impact when it reaches the bottom. But then you think, what if I jump just before that happens? When you jump, you're moving up, not down, so there won't be any impact at all!



            The mistake here is the same as the one you're made above. When you jump in the elevator, you indeed start moving upward relative to the elevator, but you're still moving at a tremendous speed downward relative to the ground, which is what matters.



            Similarly, when you are at the rim of a large rotating space station, you have a large velocity relative to somebody standing still at the center. When you jump, it's true that you're going up relative to the piece of ground you jumped from, but you still have that huge rotational velocity. You don't lose it just by losing contact with the ground, so nothing about the story changes.






            share|cite|improve this answer













            Now, the question: if, while being inside a rotating space station, astronaut would jump really high, wouldn't he then experience zero gravity until he again will touch some part (wall or floor) of the station? Am I missing something in my understanding?




            Well, here's a related question. Suppose you find yourself in an elevator at the top floor of a skyscraper when the cable suddenly snaps. As the elevator plummets down, you realize you'll die on impact when it reaches the bottom. But then you think, what if I jump just before that happens? When you jump, you're moving up, not down, so there won't be any impact at all!



            The mistake here is the same as the one you're made above. When you jump in the elevator, you indeed start moving upward relative to the elevator, but you're still moving at a tremendous speed downward relative to the ground, which is what matters.



            Similarly, when you are at the rim of a large rotating space station, you have a large velocity relative to somebody standing still at the center. When you jump, it's true that you're going up relative to the piece of ground you jumped from, but you still have that huge rotational velocity. You don't lose it just by losing contact with the ground, so nothing about the story changes.







            share|cite|improve this answer












            share|cite|improve this answer



            share|cite|improve this answer










            answered yesterday









            knzhou

            39.7k9111194




            39.7k9111194








            • 5




              Another, related example: A hovering helicopter on earth does not see the earth move underneath it with 1000 km/h or so (due to earth rotation), simply because that same helicopter was moving at the same speed with the earth all along.
              – user1583209
              yesterday






            • 2




              Please do correct me if I am wrong, but I think here what happens in your explanation is that although you do land on the same spot of the disc after jumping off, it’s less due to the efffect of gravity and more due to the ring rotating to ‘catch’ you. Although you do have a horizontal velocity, when you jump off I think you don’t actually move in a circle, rather you move in a straight line until hitting the ring again. In that sense, OP is technically correct since you aren’t really(assumed gravity negligible) under the effect of some centrifugal force when you jump?
              – EigenFunction
              yesterday






            • 6




              @EigenFunction You definitely are under such a force in the rotating frame of reference of the rim of the station. True, an external observer would see you move in a straight line and intersect the rim again with no centrifugal force involved, but that observer never sees a centrifugal force--only the centripetal normal force of the station's rim on your feet.
              – eyeballfrog
              yesterday








            • 7




              @EigenFunction You could always say that, though. For example, you could say that gravity on Earth doesn't really exist: a thrown ball always goes in a straight line, but the floor is just constantly accelerating up to catch it. The fact that this is always just as good a description is the content of the equivalence principle, i.e. the foundation of general relativity. You just can't tell the two apart.
              – knzhou
              yesterday








            • 1




              @simple if you enter the spinning room from an axle having not touched the station you would indeed just float there while the station spins around you. If you enter that room from a rotating spoke you'll find you'll also float but you're body will spin as you float because you'll have the same angular momentum you had when you last touched the station.
              – candied_orange
              22 hours ago
















            • 5




              Another, related example: A hovering helicopter on earth does not see the earth move underneath it with 1000 km/h or so (due to earth rotation), simply because that same helicopter was moving at the same speed with the earth all along.
              – user1583209
              yesterday






            • 2




              Please do correct me if I am wrong, but I think here what happens in your explanation is that although you do land on the same spot of the disc after jumping off, it’s less due to the efffect of gravity and more due to the ring rotating to ‘catch’ you. Although you do have a horizontal velocity, when you jump off I think you don’t actually move in a circle, rather you move in a straight line until hitting the ring again. In that sense, OP is technically correct since you aren’t really(assumed gravity negligible) under the effect of some centrifugal force when you jump?
              – EigenFunction
              yesterday






            • 6




              @EigenFunction You definitely are under such a force in the rotating frame of reference of the rim of the station. True, an external observer would see you move in a straight line and intersect the rim again with no centrifugal force involved, but that observer never sees a centrifugal force--only the centripetal normal force of the station's rim on your feet.
              – eyeballfrog
              yesterday








            • 7




              @EigenFunction You could always say that, though. For example, you could say that gravity on Earth doesn't really exist: a thrown ball always goes in a straight line, but the floor is just constantly accelerating up to catch it. The fact that this is always just as good a description is the content of the equivalence principle, i.e. the foundation of general relativity. You just can't tell the two apart.
              – knzhou
              yesterday








            • 1




              @simple if you enter the spinning room from an axle having not touched the station you would indeed just float there while the station spins around you. If you enter that room from a rotating spoke you'll find you'll also float but you're body will spin as you float because you'll have the same angular momentum you had when you last touched the station.
              – candied_orange
              22 hours ago










            5




            5




            Another, related example: A hovering helicopter on earth does not see the earth move underneath it with 1000 km/h or so (due to earth rotation), simply because that same helicopter was moving at the same speed with the earth all along.
            – user1583209
            yesterday




            Another, related example: A hovering helicopter on earth does not see the earth move underneath it with 1000 km/h or so (due to earth rotation), simply because that same helicopter was moving at the same speed with the earth all along.
            – user1583209
            yesterday




            2




            2




            Please do correct me if I am wrong, but I think here what happens in your explanation is that although you do land on the same spot of the disc after jumping off, it’s less due to the efffect of gravity and more due to the ring rotating to ‘catch’ you. Although you do have a horizontal velocity, when you jump off I think you don’t actually move in a circle, rather you move in a straight line until hitting the ring again. In that sense, OP is technically correct since you aren’t really(assumed gravity negligible) under the effect of some centrifugal force when you jump?
            – EigenFunction
            yesterday




            Please do correct me if I am wrong, but I think here what happens in your explanation is that although you do land on the same spot of the disc after jumping off, it’s less due to the efffect of gravity and more due to the ring rotating to ‘catch’ you. Although you do have a horizontal velocity, when you jump off I think you don’t actually move in a circle, rather you move in a straight line until hitting the ring again. In that sense, OP is technically correct since you aren’t really(assumed gravity negligible) under the effect of some centrifugal force when you jump?
            – EigenFunction
            yesterday




            6




            6




            @EigenFunction You definitely are under such a force in the rotating frame of reference of the rim of the station. True, an external observer would see you move in a straight line and intersect the rim again with no centrifugal force involved, but that observer never sees a centrifugal force--only the centripetal normal force of the station's rim on your feet.
            – eyeballfrog
            yesterday






            @EigenFunction You definitely are under such a force in the rotating frame of reference of the rim of the station. True, an external observer would see you move in a straight line and intersect the rim again with no centrifugal force involved, but that observer never sees a centrifugal force--only the centripetal normal force of the station's rim on your feet.
            – eyeballfrog
            yesterday






            7




            7




            @EigenFunction You could always say that, though. For example, you could say that gravity on Earth doesn't really exist: a thrown ball always goes in a straight line, but the floor is just constantly accelerating up to catch it. The fact that this is always just as good a description is the content of the equivalence principle, i.e. the foundation of general relativity. You just can't tell the two apart.
            – knzhou
            yesterday






            @EigenFunction You could always say that, though. For example, you could say that gravity on Earth doesn't really exist: a thrown ball always goes in a straight line, but the floor is just constantly accelerating up to catch it. The fact that this is always just as good a description is the content of the equivalence principle, i.e. the foundation of general relativity. You just can't tell the two apart.
            – knzhou
            yesterday






            1




            1




            @simple if you enter the spinning room from an axle having not touched the station you would indeed just float there while the station spins around you. If you enter that room from a rotating spoke you'll find you'll also float but you're body will spin as you float because you'll have the same angular momentum you had when you last touched the station.
            – candied_orange
            22 hours ago






            @simple if you enter the spinning room from an axle having not touched the station you would indeed just float there while the station spins around you. If you enter that room from a rotating spoke you'll find you'll also float but you're body will spin as you float because you'll have the same angular momentum you had when you last touched the station.
            – candied_orange
            22 hours ago












            up vote
            17
            down vote













            If you jump then you are in free fall, apart from air resistance, so you are weightless. This holds for any jump. For a brief moment you experience zero gravity!






            share|cite|improve this answer

















            • 3




              You may add that unfortunately, your environment continues to move around you. (This is true even when on earth, because the gravity exerted by our planet is indistinguishable from an accelerated movement of our planet towards us. (Hello to all flat-earthers!)
              – Peter A. Schneider
              15 hours ago












            • On the other hand you could run opposite to the direction of the ships rotation, and find yourself in lower and lower gravity as you went faster. going at the same angular velocity you would indeed find yourself weightless.
              – R. Rankin
              6 hours ago















            up vote
            17
            down vote













            If you jump then you are in free fall, apart from air resistance, so you are weightless. This holds for any jump. For a brief moment you experience zero gravity!






            share|cite|improve this answer

















            • 3




              You may add that unfortunately, your environment continues to move around you. (This is true even when on earth, because the gravity exerted by our planet is indistinguishable from an accelerated movement of our planet towards us. (Hello to all flat-earthers!)
              – Peter A. Schneider
              15 hours ago












            • On the other hand you could run opposite to the direction of the ships rotation, and find yourself in lower and lower gravity as you went faster. going at the same angular velocity you would indeed find yourself weightless.
              – R. Rankin
              6 hours ago













            up vote
            17
            down vote










            up vote
            17
            down vote









            If you jump then you are in free fall, apart from air resistance, so you are weightless. This holds for any jump. For a brief moment you experience zero gravity!






            share|cite|improve this answer












            If you jump then you are in free fall, apart from air resistance, so you are weightless. This holds for any jump. For a brief moment you experience zero gravity!







            share|cite|improve this answer












            share|cite|improve this answer



            share|cite|improve this answer










            answered yesterday









            my2cts

            4,2072417




            4,2072417








            • 3




              You may add that unfortunately, your environment continues to move around you. (This is true even when on earth, because the gravity exerted by our planet is indistinguishable from an accelerated movement of our planet towards us. (Hello to all flat-earthers!)
              – Peter A. Schneider
              15 hours ago












            • On the other hand you could run opposite to the direction of the ships rotation, and find yourself in lower and lower gravity as you went faster. going at the same angular velocity you would indeed find yourself weightless.
              – R. Rankin
              6 hours ago














            • 3




              You may add that unfortunately, your environment continues to move around you. (This is true even when on earth, because the gravity exerted by our planet is indistinguishable from an accelerated movement of our planet towards us. (Hello to all flat-earthers!)
              – Peter A. Schneider
              15 hours ago












            • On the other hand you could run opposite to the direction of the ships rotation, and find yourself in lower and lower gravity as you went faster. going at the same angular velocity you would indeed find yourself weightless.
              – R. Rankin
              6 hours ago








            3




            3




            You may add that unfortunately, your environment continues to move around you. (This is true even when on earth, because the gravity exerted by our planet is indistinguishable from an accelerated movement of our planet towards us. (Hello to all flat-earthers!)
            – Peter A. Schneider
            15 hours ago






            You may add that unfortunately, your environment continues to move around you. (This is true even when on earth, because the gravity exerted by our planet is indistinguishable from an accelerated movement of our planet towards us. (Hello to all flat-earthers!)
            – Peter A. Schneider
            15 hours ago














            On the other hand you could run opposite to the direction of the ships rotation, and find yourself in lower and lower gravity as you went faster. going at the same angular velocity you would indeed find yourself weightless.
            – R. Rankin
            6 hours ago




            On the other hand you could run opposite to the direction of the ships rotation, and find yourself in lower and lower gravity as you went faster. going at the same angular velocity you would indeed find yourself weightless.
            – R. Rankin
            6 hours ago










            up vote
            3
            down vote













            This is an inertia problem. The phrasing of your question presupposes that the astronaut loses inertia when jumping, resulting in their upward trajectory being the only major acting force. At any point while rotating in a stationary position, the inertia of the astronaut is outwards from the rotational path at a 90 degree angle from the line connecting the astronaut and the axis of rotation. When the astronaut jumps, this inertia is maintained and two new forces are generated, namely the force the astronaut enacts on the space station (resulting in negligible acceleration on the part of the space station) and the force the space station enacts on the astronaut (resulting in non-negligible acceleration on the part of the astronaut towards the axis of rotation). The astronaut's relative velocity is related linearly to the vector sum of the inertial force acting on him as well as the force from his jump. This is why the "gravity" felt by the astronaut is determined by the angular velocity of the station floor; greater angular velocity means greater inertia as well as greater centripetal (and centrifugal) force, which means greater force (applied by jumping) is required to accomplish the same (relatively) vertical displacement.



            The feeling of being pressed to the ground in an artificial gravity chamber is due to the rotating floor pushing the astronaut towards the rotational axis, when the astronaut (from a physics perspective) wants to keep going along the inertial path. This force resulting in this central acceleration is the centripetal force, while the opposing force enacted by the astronaut on the space station floor is the centrifugal force.



            You can find this information in any basic university level physics book; pay special attention to chapters covering angular momentum and gravitational/circular forces. Fundamentals of Physics by David Halliday and Robert Resnick was the one I used in college.






            share|cite|improve this answer








            New contributor




            SweepingsDemon is a new contributor to this site. Take care in asking for clarification, commenting, and answering.
            Check out our Code of Conduct.






















              up vote
              3
              down vote













              This is an inertia problem. The phrasing of your question presupposes that the astronaut loses inertia when jumping, resulting in their upward trajectory being the only major acting force. At any point while rotating in a stationary position, the inertia of the astronaut is outwards from the rotational path at a 90 degree angle from the line connecting the astronaut and the axis of rotation. When the astronaut jumps, this inertia is maintained and two new forces are generated, namely the force the astronaut enacts on the space station (resulting in negligible acceleration on the part of the space station) and the force the space station enacts on the astronaut (resulting in non-negligible acceleration on the part of the astronaut towards the axis of rotation). The astronaut's relative velocity is related linearly to the vector sum of the inertial force acting on him as well as the force from his jump. This is why the "gravity" felt by the astronaut is determined by the angular velocity of the station floor; greater angular velocity means greater inertia as well as greater centripetal (and centrifugal) force, which means greater force (applied by jumping) is required to accomplish the same (relatively) vertical displacement.



              The feeling of being pressed to the ground in an artificial gravity chamber is due to the rotating floor pushing the astronaut towards the rotational axis, when the astronaut (from a physics perspective) wants to keep going along the inertial path. This force resulting in this central acceleration is the centripetal force, while the opposing force enacted by the astronaut on the space station floor is the centrifugal force.



              You can find this information in any basic university level physics book; pay special attention to chapters covering angular momentum and gravitational/circular forces. Fundamentals of Physics by David Halliday and Robert Resnick was the one I used in college.






              share|cite|improve this answer








              New contributor




              SweepingsDemon is a new contributor to this site. Take care in asking for clarification, commenting, and answering.
              Check out our Code of Conduct.




















                up vote
                3
                down vote










                up vote
                3
                down vote









                This is an inertia problem. The phrasing of your question presupposes that the astronaut loses inertia when jumping, resulting in their upward trajectory being the only major acting force. At any point while rotating in a stationary position, the inertia of the astronaut is outwards from the rotational path at a 90 degree angle from the line connecting the astronaut and the axis of rotation. When the astronaut jumps, this inertia is maintained and two new forces are generated, namely the force the astronaut enacts on the space station (resulting in negligible acceleration on the part of the space station) and the force the space station enacts on the astronaut (resulting in non-negligible acceleration on the part of the astronaut towards the axis of rotation). The astronaut's relative velocity is related linearly to the vector sum of the inertial force acting on him as well as the force from his jump. This is why the "gravity" felt by the astronaut is determined by the angular velocity of the station floor; greater angular velocity means greater inertia as well as greater centripetal (and centrifugal) force, which means greater force (applied by jumping) is required to accomplish the same (relatively) vertical displacement.



                The feeling of being pressed to the ground in an artificial gravity chamber is due to the rotating floor pushing the astronaut towards the rotational axis, when the astronaut (from a physics perspective) wants to keep going along the inertial path. This force resulting in this central acceleration is the centripetal force, while the opposing force enacted by the astronaut on the space station floor is the centrifugal force.



                You can find this information in any basic university level physics book; pay special attention to chapters covering angular momentum and gravitational/circular forces. Fundamentals of Physics by David Halliday and Robert Resnick was the one I used in college.






                share|cite|improve this answer








                New contributor




                SweepingsDemon is a new contributor to this site. Take care in asking for clarification, commenting, and answering.
                Check out our Code of Conduct.









                This is an inertia problem. The phrasing of your question presupposes that the astronaut loses inertia when jumping, resulting in their upward trajectory being the only major acting force. At any point while rotating in a stationary position, the inertia of the astronaut is outwards from the rotational path at a 90 degree angle from the line connecting the astronaut and the axis of rotation. When the astronaut jumps, this inertia is maintained and two new forces are generated, namely the force the astronaut enacts on the space station (resulting in negligible acceleration on the part of the space station) and the force the space station enacts on the astronaut (resulting in non-negligible acceleration on the part of the astronaut towards the axis of rotation). The astronaut's relative velocity is related linearly to the vector sum of the inertial force acting on him as well as the force from his jump. This is why the "gravity" felt by the astronaut is determined by the angular velocity of the station floor; greater angular velocity means greater inertia as well as greater centripetal (and centrifugal) force, which means greater force (applied by jumping) is required to accomplish the same (relatively) vertical displacement.



                The feeling of being pressed to the ground in an artificial gravity chamber is due to the rotating floor pushing the astronaut towards the rotational axis, when the astronaut (from a physics perspective) wants to keep going along the inertial path. This force resulting in this central acceleration is the centripetal force, while the opposing force enacted by the astronaut on the space station floor is the centrifugal force.



                You can find this information in any basic university level physics book; pay special attention to chapters covering angular momentum and gravitational/circular forces. Fundamentals of Physics by David Halliday and Robert Resnick was the one I used in college.







                share|cite|improve this answer








                New contributor




                SweepingsDemon is a new contributor to this site. Take care in asking for clarification, commenting, and answering.
                Check out our Code of Conduct.









                share|cite|improve this answer



                share|cite|improve this answer






                New contributor




                SweepingsDemon is a new contributor to this site. Take care in asking for clarification, commenting, and answering.
                Check out our Code of Conduct.









                answered yesterday









                SweepingsDemon

                311




                311




                New contributor




                SweepingsDemon is a new contributor to this site. Take care in asking for clarification, commenting, and answering.
                Check out our Code of Conduct.





                New contributor





                SweepingsDemon is a new contributor to this site. Take care in asking for clarification, commenting, and answering.
                Check out our Code of Conduct.






                SweepingsDemon is a new contributor to this site. Take care in asking for clarification, commenting, and answering.
                Check out our Code of Conduct.






















                    up vote
                    1
                    down vote













                    Of course, the biggest problem with the supposition of becoming 'weightless' (returning to a null-G state) by making a 'big' jump is this: Assuming the OP was talking about not a created micro-gravity (.2G or less) but closer to one full gravity (Earth standard), is the size of the 'room' required to create that much tidal force. The smaller it is, the faster it would have to be spinning to create sufficient centripetal force to give the impression of that 'gravity', and subsequently the thinner the band of that 'gravity' as the further from the zone of that 'gravity' the less the force.



                    So unless it were some silly, tiny spinning room, the size of the 'station' required to create a 'natural gravitational' feel would far exceed the astronaut's ability to jump without outside propulsion. In that case, of course, the ability to then travel outside the functional 'gravity' into decreasingly accelerated zones (by removing his own laterally imparted velocity) he could then eventually reach the 0G center of the station or room.



                    However, with the additional concept of scale, the same could be said of the Earth itself. If you could 'jump' to a 'high enough' height, you could reach 0G (not really, but sufficient micro gravity as to be indistinguishable from 0G). This does give pause for thought, as not only would such a station need to be QUITE large, but that you'd better be wearing a low-pressure suit at the same time, as the atmosphere should be quite thin, since the Nitrogen/Oxygen atmosphere would ALSO be affected by the centripetal force of the station's rotation, thus not only be thin, but definitely not made up of the proper gasses for breathing (lighter gasses rising into the center of spin)... At least that's what I'd expect.






                    share|cite|improve this answer








                    New contributor




                    Asuka Jr. is a new contributor to this site. Take care in asking for clarification, commenting, and answering.
                    Check out our Code of Conduct.






















                      up vote
                      1
                      down vote













                      Of course, the biggest problem with the supposition of becoming 'weightless' (returning to a null-G state) by making a 'big' jump is this: Assuming the OP was talking about not a created micro-gravity (.2G or less) but closer to one full gravity (Earth standard), is the size of the 'room' required to create that much tidal force. The smaller it is, the faster it would have to be spinning to create sufficient centripetal force to give the impression of that 'gravity', and subsequently the thinner the band of that 'gravity' as the further from the zone of that 'gravity' the less the force.



                      So unless it were some silly, tiny spinning room, the size of the 'station' required to create a 'natural gravitational' feel would far exceed the astronaut's ability to jump without outside propulsion. In that case, of course, the ability to then travel outside the functional 'gravity' into decreasingly accelerated zones (by removing his own laterally imparted velocity) he could then eventually reach the 0G center of the station or room.



                      However, with the additional concept of scale, the same could be said of the Earth itself. If you could 'jump' to a 'high enough' height, you could reach 0G (not really, but sufficient micro gravity as to be indistinguishable from 0G). This does give pause for thought, as not only would such a station need to be QUITE large, but that you'd better be wearing a low-pressure suit at the same time, as the atmosphere should be quite thin, since the Nitrogen/Oxygen atmosphere would ALSO be affected by the centripetal force of the station's rotation, thus not only be thin, but definitely not made up of the proper gasses for breathing (lighter gasses rising into the center of spin)... At least that's what I'd expect.






                      share|cite|improve this answer








                      New contributor




                      Asuka Jr. is a new contributor to this site. Take care in asking for clarification, commenting, and answering.
                      Check out our Code of Conduct.




















                        up vote
                        1
                        down vote










                        up vote
                        1
                        down vote









                        Of course, the biggest problem with the supposition of becoming 'weightless' (returning to a null-G state) by making a 'big' jump is this: Assuming the OP was talking about not a created micro-gravity (.2G or less) but closer to one full gravity (Earth standard), is the size of the 'room' required to create that much tidal force. The smaller it is, the faster it would have to be spinning to create sufficient centripetal force to give the impression of that 'gravity', and subsequently the thinner the band of that 'gravity' as the further from the zone of that 'gravity' the less the force.



                        So unless it were some silly, tiny spinning room, the size of the 'station' required to create a 'natural gravitational' feel would far exceed the astronaut's ability to jump without outside propulsion. In that case, of course, the ability to then travel outside the functional 'gravity' into decreasingly accelerated zones (by removing his own laterally imparted velocity) he could then eventually reach the 0G center of the station or room.



                        However, with the additional concept of scale, the same could be said of the Earth itself. If you could 'jump' to a 'high enough' height, you could reach 0G (not really, but sufficient micro gravity as to be indistinguishable from 0G). This does give pause for thought, as not only would such a station need to be QUITE large, but that you'd better be wearing a low-pressure suit at the same time, as the atmosphere should be quite thin, since the Nitrogen/Oxygen atmosphere would ALSO be affected by the centripetal force of the station's rotation, thus not only be thin, but definitely not made up of the proper gasses for breathing (lighter gasses rising into the center of spin)... At least that's what I'd expect.






                        share|cite|improve this answer








                        New contributor




                        Asuka Jr. is a new contributor to this site. Take care in asking for clarification, commenting, and answering.
                        Check out our Code of Conduct.









                        Of course, the biggest problem with the supposition of becoming 'weightless' (returning to a null-G state) by making a 'big' jump is this: Assuming the OP was talking about not a created micro-gravity (.2G or less) but closer to one full gravity (Earth standard), is the size of the 'room' required to create that much tidal force. The smaller it is, the faster it would have to be spinning to create sufficient centripetal force to give the impression of that 'gravity', and subsequently the thinner the band of that 'gravity' as the further from the zone of that 'gravity' the less the force.



                        So unless it were some silly, tiny spinning room, the size of the 'station' required to create a 'natural gravitational' feel would far exceed the astronaut's ability to jump without outside propulsion. In that case, of course, the ability to then travel outside the functional 'gravity' into decreasingly accelerated zones (by removing his own laterally imparted velocity) he could then eventually reach the 0G center of the station or room.



                        However, with the additional concept of scale, the same could be said of the Earth itself. If you could 'jump' to a 'high enough' height, you could reach 0G (not really, but sufficient micro gravity as to be indistinguishable from 0G). This does give pause for thought, as not only would such a station need to be QUITE large, but that you'd better be wearing a low-pressure suit at the same time, as the atmosphere should be quite thin, since the Nitrogen/Oxygen atmosphere would ALSO be affected by the centripetal force of the station's rotation, thus not only be thin, but definitely not made up of the proper gasses for breathing (lighter gasses rising into the center of spin)... At least that's what I'd expect.







                        share|cite|improve this answer








                        New contributor




                        Asuka Jr. is a new contributor to this site. Take care in asking for clarification, commenting, and answering.
                        Check out our Code of Conduct.









                        share|cite|improve this answer



                        share|cite|improve this answer






                        New contributor




                        Asuka Jr. is a new contributor to this site. Take care in asking for clarification, commenting, and answering.
                        Check out our Code of Conduct.









                        answered 22 hours ago









                        Asuka Jr.

                        111




                        111




                        New contributor




                        Asuka Jr. is a new contributor to this site. Take care in asking for clarification, commenting, and answering.
                        Check out our Code of Conduct.





                        New contributor





                        Asuka Jr. is a new contributor to this site. Take care in asking for clarification, commenting, and answering.
                        Check out our Code of Conduct.






                        Asuka Jr. is a new contributor to this site. Take care in asking for clarification, commenting, and answering.
                        Check out our Code of Conduct.






















                            up vote
                            1
                            down vote














                            As I understand it, in order for the centrifugal force (which is responsible for creating gravity, in this case) to work, object it works upon should be attached to the wheels 'spoke' or 'rim'.




                            Not exactly. Centrifugal "force" is what's known as a "pseudo-force". It's the result of analyzing events using a non-inertial reference frame. If you're on a merry-go-round, and you treat the merry-go-round as being stationary while the world revolves around it, you will find that objects have a tendency to go towards the outside of the merry-go-round. So within the "The merry-go-round is stationary" point view, you have to posit some force pushing the objects away from the center, which is the centrifugal force.



                            But this force doesn't "really" exist: when you analyze the situation from the point of view of someone not on the merry-go-round, the objects are traveling in a straight line. It's just that any straight line will necessarily go away from the center. (Draw a circle, then draw straight line next to it. Imagine traveling along that line. From the point of view of the circle, you're first getting closer, then traveling away.)



                            When something is rotating, its velocity is constantly changing: although its speed is constant, the direction is changing, so the velocity is changing. Changing velocity means acceleration, and acceleration means force. This force is directed towards the center. Imagine driving around a circle counterclockwise. If you were to let go of the wheel, you would fly off the circle. You have to constantly turn left to stay on the circle. So there is a force, but it's towards the center of the circle, and is called the centripetal force.



                            From an inertial reference frame pint of view, a force is needed to stay on the circle; the centripetal force. But from a circular motion reference frame, the object is stationary. So if there's a centripetal force pulling the object in, there must be another force, the centrifugal force, pushing it out. So if you're standing in a rotating space station, you'll going to feel a force of the floor pushing you "up" towards the center of the space station, and since it feels like you're at rest (the space station is moving with you), it's going to seem that there must be some force pushing you "down" into the floor.



                            The important point here is that the contact with the floor provides the centripetal force, but the centrifugal force exists in your reference frame regardless of whether you have contact with the floor. Go back to the example of driving in a circle. Suppose you drop a ball in the car. Before you dropped it, it was moving with the car, and so just as the car had a centripetal force keeping in circular motion, the ball had a centripetal force on it. But for the fraction of a second that it's in the air, it doesn't have the centripetal force.



                            For an outside observer, the car is turning left, while the ball is moving in a straight line. The car accelerates to the left into the ball, and when the ball lands, it is to the right of where it was dropped. For someone in the car, however, it seems like the car is stationary, and ball is accelerating to the right.



                            Similarly, if you were to jump in the space station, then since you are perceiving things in the space station's reference frame, it will seem like you are accelerating towards the floor. This apparent acceleration exists regardless of whether you're touching the space station. That you are accelerating in the station's reference frame doesn't require physical contact with the station because it's not a physical phenomenon. It's simply an attribute of the coordinate system.



                            All of this applies locally: if you jump up, your motion will, in the station's reference frame, on small scales be the same as if you were being pulled down by gravity. This is an approximation that breaks down as you go to larger scales. These deviations from the approximation show up as other pseudo-forces, such as the Coriolis force. So being in contact with the floor does matter in that it keeps you moving with the station and reduces these deviations.






                            share|cite|improve this answer





















                            • Maybe worth emphasising the Coriolis force a bit more, and describing the case where someone jumps backwards fast enough to cancel the rotation and so the centrifugal force would still exist but would be exactly balanced by coriolis so they would appear to fly above the floor without falling. (until they hit a wall)
                              – bdsl
                              10 hours ago















                            up vote
                            1
                            down vote














                            As I understand it, in order for the centrifugal force (which is responsible for creating gravity, in this case) to work, object it works upon should be attached to the wheels 'spoke' or 'rim'.




                            Not exactly. Centrifugal "force" is what's known as a "pseudo-force". It's the result of analyzing events using a non-inertial reference frame. If you're on a merry-go-round, and you treat the merry-go-round as being stationary while the world revolves around it, you will find that objects have a tendency to go towards the outside of the merry-go-round. So within the "The merry-go-round is stationary" point view, you have to posit some force pushing the objects away from the center, which is the centrifugal force.



                            But this force doesn't "really" exist: when you analyze the situation from the point of view of someone not on the merry-go-round, the objects are traveling in a straight line. It's just that any straight line will necessarily go away from the center. (Draw a circle, then draw straight line next to it. Imagine traveling along that line. From the point of view of the circle, you're first getting closer, then traveling away.)



                            When something is rotating, its velocity is constantly changing: although its speed is constant, the direction is changing, so the velocity is changing. Changing velocity means acceleration, and acceleration means force. This force is directed towards the center. Imagine driving around a circle counterclockwise. If you were to let go of the wheel, you would fly off the circle. You have to constantly turn left to stay on the circle. So there is a force, but it's towards the center of the circle, and is called the centripetal force.



                            From an inertial reference frame pint of view, a force is needed to stay on the circle; the centripetal force. But from a circular motion reference frame, the object is stationary. So if there's a centripetal force pulling the object in, there must be another force, the centrifugal force, pushing it out. So if you're standing in a rotating space station, you'll going to feel a force of the floor pushing you "up" towards the center of the space station, and since it feels like you're at rest (the space station is moving with you), it's going to seem that there must be some force pushing you "down" into the floor.



                            The important point here is that the contact with the floor provides the centripetal force, but the centrifugal force exists in your reference frame regardless of whether you have contact with the floor. Go back to the example of driving in a circle. Suppose you drop a ball in the car. Before you dropped it, it was moving with the car, and so just as the car had a centripetal force keeping in circular motion, the ball had a centripetal force on it. But for the fraction of a second that it's in the air, it doesn't have the centripetal force.



                            For an outside observer, the car is turning left, while the ball is moving in a straight line. The car accelerates to the left into the ball, and when the ball lands, it is to the right of where it was dropped. For someone in the car, however, it seems like the car is stationary, and ball is accelerating to the right.



                            Similarly, if you were to jump in the space station, then since you are perceiving things in the space station's reference frame, it will seem like you are accelerating towards the floor. This apparent acceleration exists regardless of whether you're touching the space station. That you are accelerating in the station's reference frame doesn't require physical contact with the station because it's not a physical phenomenon. It's simply an attribute of the coordinate system.



                            All of this applies locally: if you jump up, your motion will, in the station's reference frame, on small scales be the same as if you were being pulled down by gravity. This is an approximation that breaks down as you go to larger scales. These deviations from the approximation show up as other pseudo-forces, such as the Coriolis force. So being in contact with the floor does matter in that it keeps you moving with the station and reduces these deviations.






                            share|cite|improve this answer





















                            • Maybe worth emphasising the Coriolis force a bit more, and describing the case where someone jumps backwards fast enough to cancel the rotation and so the centrifugal force would still exist but would be exactly balanced by coriolis so they would appear to fly above the floor without falling. (until they hit a wall)
                              – bdsl
                              10 hours ago













                            up vote
                            1
                            down vote










                            up vote
                            1
                            down vote










                            As I understand it, in order for the centrifugal force (which is responsible for creating gravity, in this case) to work, object it works upon should be attached to the wheels 'spoke' or 'rim'.




                            Not exactly. Centrifugal "force" is what's known as a "pseudo-force". It's the result of analyzing events using a non-inertial reference frame. If you're on a merry-go-round, and you treat the merry-go-round as being stationary while the world revolves around it, you will find that objects have a tendency to go towards the outside of the merry-go-round. So within the "The merry-go-round is stationary" point view, you have to posit some force pushing the objects away from the center, which is the centrifugal force.



                            But this force doesn't "really" exist: when you analyze the situation from the point of view of someone not on the merry-go-round, the objects are traveling in a straight line. It's just that any straight line will necessarily go away from the center. (Draw a circle, then draw straight line next to it. Imagine traveling along that line. From the point of view of the circle, you're first getting closer, then traveling away.)



                            When something is rotating, its velocity is constantly changing: although its speed is constant, the direction is changing, so the velocity is changing. Changing velocity means acceleration, and acceleration means force. This force is directed towards the center. Imagine driving around a circle counterclockwise. If you were to let go of the wheel, you would fly off the circle. You have to constantly turn left to stay on the circle. So there is a force, but it's towards the center of the circle, and is called the centripetal force.



                            From an inertial reference frame pint of view, a force is needed to stay on the circle; the centripetal force. But from a circular motion reference frame, the object is stationary. So if there's a centripetal force pulling the object in, there must be another force, the centrifugal force, pushing it out. So if you're standing in a rotating space station, you'll going to feel a force of the floor pushing you "up" towards the center of the space station, and since it feels like you're at rest (the space station is moving with you), it's going to seem that there must be some force pushing you "down" into the floor.



                            The important point here is that the contact with the floor provides the centripetal force, but the centrifugal force exists in your reference frame regardless of whether you have contact with the floor. Go back to the example of driving in a circle. Suppose you drop a ball in the car. Before you dropped it, it was moving with the car, and so just as the car had a centripetal force keeping in circular motion, the ball had a centripetal force on it. But for the fraction of a second that it's in the air, it doesn't have the centripetal force.



                            For an outside observer, the car is turning left, while the ball is moving in a straight line. The car accelerates to the left into the ball, and when the ball lands, it is to the right of where it was dropped. For someone in the car, however, it seems like the car is stationary, and ball is accelerating to the right.



                            Similarly, if you were to jump in the space station, then since you are perceiving things in the space station's reference frame, it will seem like you are accelerating towards the floor. This apparent acceleration exists regardless of whether you're touching the space station. That you are accelerating in the station's reference frame doesn't require physical contact with the station because it's not a physical phenomenon. It's simply an attribute of the coordinate system.



                            All of this applies locally: if you jump up, your motion will, in the station's reference frame, on small scales be the same as if you were being pulled down by gravity. This is an approximation that breaks down as you go to larger scales. These deviations from the approximation show up as other pseudo-forces, such as the Coriolis force. So being in contact with the floor does matter in that it keeps you moving with the station and reduces these deviations.






                            share|cite|improve this answer













                            As I understand it, in order for the centrifugal force (which is responsible for creating gravity, in this case) to work, object it works upon should be attached to the wheels 'spoke' or 'rim'.




                            Not exactly. Centrifugal "force" is what's known as a "pseudo-force". It's the result of analyzing events using a non-inertial reference frame. If you're on a merry-go-round, and you treat the merry-go-round as being stationary while the world revolves around it, you will find that objects have a tendency to go towards the outside of the merry-go-round. So within the "The merry-go-round is stationary" point view, you have to posit some force pushing the objects away from the center, which is the centrifugal force.



                            But this force doesn't "really" exist: when you analyze the situation from the point of view of someone not on the merry-go-round, the objects are traveling in a straight line. It's just that any straight line will necessarily go away from the center. (Draw a circle, then draw straight line next to it. Imagine traveling along that line. From the point of view of the circle, you're first getting closer, then traveling away.)



                            When something is rotating, its velocity is constantly changing: although its speed is constant, the direction is changing, so the velocity is changing. Changing velocity means acceleration, and acceleration means force. This force is directed towards the center. Imagine driving around a circle counterclockwise. If you were to let go of the wheel, you would fly off the circle. You have to constantly turn left to stay on the circle. So there is a force, but it's towards the center of the circle, and is called the centripetal force.



                            From an inertial reference frame pint of view, a force is needed to stay on the circle; the centripetal force. But from a circular motion reference frame, the object is stationary. So if there's a centripetal force pulling the object in, there must be another force, the centrifugal force, pushing it out. So if you're standing in a rotating space station, you'll going to feel a force of the floor pushing you "up" towards the center of the space station, and since it feels like you're at rest (the space station is moving with you), it's going to seem that there must be some force pushing you "down" into the floor.



                            The important point here is that the contact with the floor provides the centripetal force, but the centrifugal force exists in your reference frame regardless of whether you have contact with the floor. Go back to the example of driving in a circle. Suppose you drop a ball in the car. Before you dropped it, it was moving with the car, and so just as the car had a centripetal force keeping in circular motion, the ball had a centripetal force on it. But for the fraction of a second that it's in the air, it doesn't have the centripetal force.



                            For an outside observer, the car is turning left, while the ball is moving in a straight line. The car accelerates to the left into the ball, and when the ball lands, it is to the right of where it was dropped. For someone in the car, however, it seems like the car is stationary, and ball is accelerating to the right.



                            Similarly, if you were to jump in the space station, then since you are perceiving things in the space station's reference frame, it will seem like you are accelerating towards the floor. This apparent acceleration exists regardless of whether you're touching the space station. That you are accelerating in the station's reference frame doesn't require physical contact with the station because it's not a physical phenomenon. It's simply an attribute of the coordinate system.



                            All of this applies locally: if you jump up, your motion will, in the station's reference frame, on small scales be the same as if you were being pulled down by gravity. This is an approximation that breaks down as you go to larger scales. These deviations from the approximation show up as other pseudo-forces, such as the Coriolis force. So being in contact with the floor does matter in that it keeps you moving with the station and reduces these deviations.







                            share|cite|improve this answer












                            share|cite|improve this answer



                            share|cite|improve this answer










                            answered 10 hours ago









                            Acccumulation

                            1,678210




                            1,678210












                            • Maybe worth emphasising the Coriolis force a bit more, and describing the case where someone jumps backwards fast enough to cancel the rotation and so the centrifugal force would still exist but would be exactly balanced by coriolis so they would appear to fly above the floor without falling. (until they hit a wall)
                              – bdsl
                              10 hours ago


















                            • Maybe worth emphasising the Coriolis force a bit more, and describing the case where someone jumps backwards fast enough to cancel the rotation and so the centrifugal force would still exist but would be exactly balanced by coriolis so they would appear to fly above the floor without falling. (until they hit a wall)
                              – bdsl
                              10 hours ago
















                            Maybe worth emphasising the Coriolis force a bit more, and describing the case where someone jumps backwards fast enough to cancel the rotation and so the centrifugal force would still exist but would be exactly balanced by coriolis so they would appear to fly above the floor without falling. (until they hit a wall)
                            – bdsl
                            10 hours ago




                            Maybe worth emphasising the Coriolis force a bit more, and describing the case where someone jumps backwards fast enough to cancel the rotation and so the centrifugal force would still exist but would be exactly balanced by coriolis so they would appear to fly above the floor without falling. (until they hit a wall)
                            – bdsl
                            10 hours ago










                            up vote
                            0
                            down vote













                            You're not making any mistake except for thinking "artificial gravity" could be
                            nearly constant over a region as big as the structure itself. It really always
                            applies only to a region "much smaller" than the structure itself.



                            So, yes, a big jump (up and backwards on the wheel) could send you through the
                            middle of the wheel, where you would just float. Or, more simply, running fast
                            enough (backwards on the wheel) will cause you to levitate.



                            This non-constant variation of your "artificial gravity" in spacetime was already
                            explained as a "tidal force" in my2cts's comment.






                            share|cite|improve this answer

























                              up vote
                              0
                              down vote













                              You're not making any mistake except for thinking "artificial gravity" could be
                              nearly constant over a region as big as the structure itself. It really always
                              applies only to a region "much smaller" than the structure itself.



                              So, yes, a big jump (up and backwards on the wheel) could send you through the
                              middle of the wheel, where you would just float. Or, more simply, running fast
                              enough (backwards on the wheel) will cause you to levitate.



                              This non-constant variation of your "artificial gravity" in spacetime was already
                              explained as a "tidal force" in my2cts's comment.






                              share|cite|improve this answer























                                up vote
                                0
                                down vote










                                up vote
                                0
                                down vote









                                You're not making any mistake except for thinking "artificial gravity" could be
                                nearly constant over a region as big as the structure itself. It really always
                                applies only to a region "much smaller" than the structure itself.



                                So, yes, a big jump (up and backwards on the wheel) could send you through the
                                middle of the wheel, where you would just float. Or, more simply, running fast
                                enough (backwards on the wheel) will cause you to levitate.



                                This non-constant variation of your "artificial gravity" in spacetime was already
                                explained as a "tidal force" in my2cts's comment.






                                share|cite|improve this answer












                                You're not making any mistake except for thinking "artificial gravity" could be
                                nearly constant over a region as big as the structure itself. It really always
                                applies only to a region "much smaller" than the structure itself.



                                So, yes, a big jump (up and backwards on the wheel) could send you through the
                                middle of the wheel, where you would just float. Or, more simply, running fast
                                enough (backwards on the wheel) will cause you to levitate.



                                This non-constant variation of your "artificial gravity" in spacetime was already
                                explained as a "tidal force" in my2cts's comment.







                                share|cite|improve this answer












                                share|cite|improve this answer



                                share|cite|improve this answer










                                answered 23 hours ago









                                bobuhito

                                7341511




                                7341511






















                                    up vote
                                    0
                                    down vote













                                    Here's a really simple experiment you can perform right now.



                                    You are standing on the surface of the Earth which is rotating at, depending on your latitude, somewhere between 1000 and 1600 km/h. If you dare, stand up and jump. Did you suddenly hurdle sideways at a great speed? No. Similarly a lamp hanging from the ceiling isn't thrown violently to one the side despite not touching the ground.



                                    Your momentum is conserved. And momentum is mass x velocity. Your mass doesn't change when you jump, so we can just talk about conserving velocity.



                                    Velocity is a vector meaning it has a magnitude and a direction. Standing still you're traveling at, say, 1200 km/h going sideways, but so is the surface of the Earth and your house and the air so you don't perceive it. When you jump you retain that 1200 km/h sideways velocity and add about 2m/s upward velocity. Since everything else is moving sideways with you, all you perceive is the vertical jump.



                                    But only your linear momentum is conserved. Gravity is keeping you stuck to the rotating Earth. If you were to somehow nullify Earth's gravitational pull you'd find yourself slowly, and then more and more rapidly, seeming to rise up from the surface of the Earth. Your linear momentum is carrying you along in a straight line, but the surface of the Earth is curved. It seems like the surface is dropping away, but without gravity sticking you to the surface you'll travel in a straight line over a curved surface.



                                    This difference between the linear and angular momentum is the Coriolis force and has practical effects on Earth and on a space station. On Earth it's why storms swirl. The much smaller scale on a space station can have subtle effects if you throw a ball, shoot a gun, or even on your inner ear.






                                    share|cite|improve this answer

























                                      up vote
                                      0
                                      down vote













                                      Here's a really simple experiment you can perform right now.



                                      You are standing on the surface of the Earth which is rotating at, depending on your latitude, somewhere between 1000 and 1600 km/h. If you dare, stand up and jump. Did you suddenly hurdle sideways at a great speed? No. Similarly a lamp hanging from the ceiling isn't thrown violently to one the side despite not touching the ground.



                                      Your momentum is conserved. And momentum is mass x velocity. Your mass doesn't change when you jump, so we can just talk about conserving velocity.



                                      Velocity is a vector meaning it has a magnitude and a direction. Standing still you're traveling at, say, 1200 km/h going sideways, but so is the surface of the Earth and your house and the air so you don't perceive it. When you jump you retain that 1200 km/h sideways velocity and add about 2m/s upward velocity. Since everything else is moving sideways with you, all you perceive is the vertical jump.



                                      But only your linear momentum is conserved. Gravity is keeping you stuck to the rotating Earth. If you were to somehow nullify Earth's gravitational pull you'd find yourself slowly, and then more and more rapidly, seeming to rise up from the surface of the Earth. Your linear momentum is carrying you along in a straight line, but the surface of the Earth is curved. It seems like the surface is dropping away, but without gravity sticking you to the surface you'll travel in a straight line over a curved surface.



                                      This difference between the linear and angular momentum is the Coriolis force and has practical effects on Earth and on a space station. On Earth it's why storms swirl. The much smaller scale on a space station can have subtle effects if you throw a ball, shoot a gun, or even on your inner ear.






                                      share|cite|improve this answer























                                        up vote
                                        0
                                        down vote










                                        up vote
                                        0
                                        down vote









                                        Here's a really simple experiment you can perform right now.



                                        You are standing on the surface of the Earth which is rotating at, depending on your latitude, somewhere between 1000 and 1600 km/h. If you dare, stand up and jump. Did you suddenly hurdle sideways at a great speed? No. Similarly a lamp hanging from the ceiling isn't thrown violently to one the side despite not touching the ground.



                                        Your momentum is conserved. And momentum is mass x velocity. Your mass doesn't change when you jump, so we can just talk about conserving velocity.



                                        Velocity is a vector meaning it has a magnitude and a direction. Standing still you're traveling at, say, 1200 km/h going sideways, but so is the surface of the Earth and your house and the air so you don't perceive it. When you jump you retain that 1200 km/h sideways velocity and add about 2m/s upward velocity. Since everything else is moving sideways with you, all you perceive is the vertical jump.



                                        But only your linear momentum is conserved. Gravity is keeping you stuck to the rotating Earth. If you were to somehow nullify Earth's gravitational pull you'd find yourself slowly, and then more and more rapidly, seeming to rise up from the surface of the Earth. Your linear momentum is carrying you along in a straight line, but the surface of the Earth is curved. It seems like the surface is dropping away, but without gravity sticking you to the surface you'll travel in a straight line over a curved surface.



                                        This difference between the linear and angular momentum is the Coriolis force and has practical effects on Earth and on a space station. On Earth it's why storms swirl. The much smaller scale on a space station can have subtle effects if you throw a ball, shoot a gun, or even on your inner ear.






                                        share|cite|improve this answer












                                        Here's a really simple experiment you can perform right now.



                                        You are standing on the surface of the Earth which is rotating at, depending on your latitude, somewhere between 1000 and 1600 km/h. If you dare, stand up and jump. Did you suddenly hurdle sideways at a great speed? No. Similarly a lamp hanging from the ceiling isn't thrown violently to one the side despite not touching the ground.



                                        Your momentum is conserved. And momentum is mass x velocity. Your mass doesn't change when you jump, so we can just talk about conserving velocity.



                                        Velocity is a vector meaning it has a magnitude and a direction. Standing still you're traveling at, say, 1200 km/h going sideways, but so is the surface of the Earth and your house and the air so you don't perceive it. When you jump you retain that 1200 km/h sideways velocity and add about 2m/s upward velocity. Since everything else is moving sideways with you, all you perceive is the vertical jump.



                                        But only your linear momentum is conserved. Gravity is keeping you stuck to the rotating Earth. If you were to somehow nullify Earth's gravitational pull you'd find yourself slowly, and then more and more rapidly, seeming to rise up from the surface of the Earth. Your linear momentum is carrying you along in a straight line, but the surface of the Earth is curved. It seems like the surface is dropping away, but without gravity sticking you to the surface you'll travel in a straight line over a curved surface.



                                        This difference between the linear and angular momentum is the Coriolis force and has practical effects on Earth and on a space station. On Earth it's why storms swirl. The much smaller scale on a space station can have subtle effects if you throw a ball, shoot a gun, or even on your inner ear.







                                        share|cite|improve this answer












                                        share|cite|improve this answer



                                        share|cite|improve this answer










                                        answered 6 hours ago









                                        Schwern

                                        3,11921022




                                        3,11921022






















                                            up vote
                                            0
                                            down vote













                                            Just a simplified example scenario depicting what the angular velocity answers say:



                                            Assume a large open cylinder set up as a jogging track that is an entire rotating "Ring" of the a space station.



                                            Instead of jumping, run as fast as the track section is rotating but in the opposite direction. You will appear to lose weight as you accelerate since you will no longer have the angular momentum of the floor but something less than that.



                                            When you reach the speed the station is spinning, you will be weightless and lift off the ground (if you don't lose traction first). While above the ground in this way you will be weightless.



                                            This also means, intuitively, that if you ran with the spin of the station you would get heavier.



                                            Once you see it this way, you can see that jumping wouldn't really do anything in itself.



                                            tl;dr



                                            Problem 1:
                                            After you become weightless the air is probably rotating with the floor of the track, so it will push your body giving it some angular momentum again at which point you will be "Pulled" back to the surface. If being able to alter your weight was a partial goal of this exercise room they could make the floor and walls fairly smooth so that the air stayed (mostly) in place and ignored the spin altogether. If this were the case you'd start with a wind at your back, but the faster you ran the weaker the wind would become and when you became weightless there would be no air pushing you.



                                            Problem 2: In order to run the speed of the station the starting gravity may have to be less than 1g, Not sure of the math at all, but I am sure with the right starting speed you could run until you were weightless.






                                            share|cite|improve this answer



























                                              up vote
                                              0
                                              down vote













                                              Just a simplified example scenario depicting what the angular velocity answers say:



                                              Assume a large open cylinder set up as a jogging track that is an entire rotating "Ring" of the a space station.



                                              Instead of jumping, run as fast as the track section is rotating but in the opposite direction. You will appear to lose weight as you accelerate since you will no longer have the angular momentum of the floor but something less than that.



                                              When you reach the speed the station is spinning, you will be weightless and lift off the ground (if you don't lose traction first). While above the ground in this way you will be weightless.



                                              This also means, intuitively, that if you ran with the spin of the station you would get heavier.



                                              Once you see it this way, you can see that jumping wouldn't really do anything in itself.



                                              tl;dr



                                              Problem 1:
                                              After you become weightless the air is probably rotating with the floor of the track, so it will push your body giving it some angular momentum again at which point you will be "Pulled" back to the surface. If being able to alter your weight was a partial goal of this exercise room they could make the floor and walls fairly smooth so that the air stayed (mostly) in place and ignored the spin altogether. If this were the case you'd start with a wind at your back, but the faster you ran the weaker the wind would become and when you became weightless there would be no air pushing you.



                                              Problem 2: In order to run the speed of the station the starting gravity may have to be less than 1g, Not sure of the math at all, but I am sure with the right starting speed you could run until you were weightless.






                                              share|cite|improve this answer

























                                                up vote
                                                0
                                                down vote










                                                up vote
                                                0
                                                down vote









                                                Just a simplified example scenario depicting what the angular velocity answers say:



                                                Assume a large open cylinder set up as a jogging track that is an entire rotating "Ring" of the a space station.



                                                Instead of jumping, run as fast as the track section is rotating but in the opposite direction. You will appear to lose weight as you accelerate since you will no longer have the angular momentum of the floor but something less than that.



                                                When you reach the speed the station is spinning, you will be weightless and lift off the ground (if you don't lose traction first). While above the ground in this way you will be weightless.



                                                This also means, intuitively, that if you ran with the spin of the station you would get heavier.



                                                Once you see it this way, you can see that jumping wouldn't really do anything in itself.



                                                tl;dr



                                                Problem 1:
                                                After you become weightless the air is probably rotating with the floor of the track, so it will push your body giving it some angular momentum again at which point you will be "Pulled" back to the surface. If being able to alter your weight was a partial goal of this exercise room they could make the floor and walls fairly smooth so that the air stayed (mostly) in place and ignored the spin altogether. If this were the case you'd start with a wind at your back, but the faster you ran the weaker the wind would become and when you became weightless there would be no air pushing you.



                                                Problem 2: In order to run the speed of the station the starting gravity may have to be less than 1g, Not sure of the math at all, but I am sure with the right starting speed you could run until you were weightless.






                                                share|cite|improve this answer














                                                Just a simplified example scenario depicting what the angular velocity answers say:



                                                Assume a large open cylinder set up as a jogging track that is an entire rotating "Ring" of the a space station.



                                                Instead of jumping, run as fast as the track section is rotating but in the opposite direction. You will appear to lose weight as you accelerate since you will no longer have the angular momentum of the floor but something less than that.



                                                When you reach the speed the station is spinning, you will be weightless and lift off the ground (if you don't lose traction first). While above the ground in this way you will be weightless.



                                                This also means, intuitively, that if you ran with the spin of the station you would get heavier.



                                                Once you see it this way, you can see that jumping wouldn't really do anything in itself.



                                                tl;dr



                                                Problem 1:
                                                After you become weightless the air is probably rotating with the floor of the track, so it will push your body giving it some angular momentum again at which point you will be "Pulled" back to the surface. If being able to alter your weight was a partial goal of this exercise room they could make the floor and walls fairly smooth so that the air stayed (mostly) in place and ignored the spin altogether. If this were the case you'd start with a wind at your back, but the faster you ran the weaker the wind would become and when you became weightless there would be no air pushing you.



                                                Problem 2: In order to run the speed of the station the starting gravity may have to be less than 1g, Not sure of the math at all, but I am sure with the right starting speed you could run until you were weightless.







                                                share|cite|improve this answer














                                                share|cite|improve this answer



                                                share|cite|improve this answer








                                                edited 6 hours ago

























                                                answered 6 hours ago









                                                Bill K

                                                1215




                                                1215






























                                                    draft saved

                                                    draft discarded




















































                                                    Thanks for contributing an answer to Physics Stack Exchange!


                                                    • Please be sure to answer the question. Provide details and share your research!

                                                    But avoid



                                                    • Asking for help, clarification, or responding to other answers.

                                                    • Making statements based on opinion; back them up with references or personal experience.


                                                    Use MathJax to format equations. MathJax reference.


                                                    To learn more, see our tips on writing great answers.





                                                    Some of your past answers have not been well-received, and you're in danger of being blocked from answering.


                                                    Please pay close attention to the following guidance:


                                                    • Please be sure to answer the question. Provide details and share your research!

                                                    But avoid



                                                    • Asking for help, clarification, or responding to other answers.

                                                    • Making statements based on opinion; back them up with references or personal experience.


                                                    To learn more, see our tips on writing great answers.




                                                    draft saved


                                                    draft discarded














                                                    StackExchange.ready(
                                                    function () {
                                                    StackExchange.openid.initPostLogin('.new-post-login', 'https%3a%2f%2fphysics.stackexchange.com%2fquestions%2f446889%2fdoes-artificial-gravity-based-on-centrifugal-force-stop-working-if-you-jump-off%23new-answer', 'question_page');
                                                    }
                                                    );

                                                    Post as a guest















                                                    Required, but never shown





















































                                                    Required, but never shown














                                                    Required, but never shown












                                                    Required, but never shown







                                                    Required, but never shown

































                                                    Required, but never shown














                                                    Required, but never shown












                                                    Required, but never shown







                                                    Required, but never shown







                                                    Popular posts from this blog

                                                    Plaza Victoria

                                                    In PowerPoint, is there a keyboard shortcut for bulleted / numbered list?

                                                    How to put 3 figures in Latex with 2 figures side by side and 1 below these side by side images but in...